Questions tagged [number-theory]

Questions on advanced topics - beyond those in typical introductory courses: higher degree algebraic number and function fields, Diophantine equations, geometry of numbers / lattices, quadratic forms, discontinuous groups and and automorphic forms, Diophantine approximation, transcendental numbers, elliptic curves and arithmetic algebraic geometry, exponential and character sums, Zeta and L-functions, multiplicative and additive number theory, etc.

Filter by
Sorted by
Tagged with
2 votes
2 answers
1k views

$a_n=2^n+3^n+6^n-1$. Find all positive integers that are primes to all terms of the sequence.

Let the sequence $a_n=2^n+3^n+6^n-1, n\in\mathbb N_{> 0}$. Find all positive integers that are prime to all terms of this sequence. I have no idea how to approach this, but I know that I CAN'T use ...
UserX's user avatar
  • 4,930
2 votes
3 answers
270 views

It is possible to show square as sum n distinc squares, where n > 2?

We know that $5^2 = 3^2 + 4^2$. But it is possible to show $a^2$ as the sum of three or more distinct squares? Something like that exist? $a^2 = b^2 + c^2 + d^2$, where $b$, $c$, $d$ are distinct ...
Nelliusz Frącek's user avatar
2 votes
2 answers
222 views

How find this minimum of the $q$, if such $\frac{95}{36}>\frac{p}{q}>\frac{96}{37}$

let $p,q$ is postive integer,and such $$\dfrac{95}{36}>\dfrac{p}{q}>\dfrac{96}{37}$$ Find the minimum of the $q$ maybe can use $$95q>36p$$ and $$37p>96q$$ and then find this minimum of ...
math110's user avatar
  • 93.4k
2 votes
3 answers
6k views

remainder when 67896789...(300 digits) divided by 999

What is the remainder when 678967896789... (300 digits)is divided by 999? i tried to divide it manually to find some pattern in remainder. But was getting bit lengthy. so please suggest me some short ...
abhisek gupta's user avatar
2 votes
2 answers
401 views

Integer solutions of the equation: $x^2+y^2+z^2=kxyz$

Given the equation: $$x^2+y^2+z^2=kxyz$$ with: $(k,x,y,z)\in\mathbb{N}$, the only solution for $k=2$ is: $x=0,y=0,z=0$. For what values of $k$ the equations has solutions in which $x,y,z$ are ...
Riccardo.Alestra's user avatar
2 votes
3 answers
142 views

Trying to prove that there are no p and q such that $|\sqrt5 - p/q| < 1/(7q^2)$.

Like the title says, I'm having trouble proving that there are no integers p and q such that $|\sqrt5 - p/q| < 1/(7q^2)$. I was given the hint that $|(q\sqrt5 - p)(q\sqrt5 + p)| \geq 1$, but I don'...
user121937's user avatar
2 votes
1 answer
676 views

What are some of the more efficient ways of studying for an Olympiad?

This September I am participating in a competition called the Australian Intermediate Mathematics olympiad, and you may not have heard of it but it's very similar to the AIME. Could you please tell me ...
user avatar
2 votes
2 answers
285 views

What does $conclude$ mean in this sentence?

My friend asked me a question, but I don't know the meaning of the sentence Conclude that $\sigma_n$ is a ring automorphism here, does it mean Prove that $\sigma_n$ is a ring automorphism or Make the ...
lsr314's user avatar
  • 15.8k
2 votes
4 answers
148 views

Is this a true theorem? [closed]

I'm trying to prove the existence of the following theorem: If $n,p \in \mathbb{N}$, then $(p+1)^n = 1 \mod p$ Is this theorem true? I think it is, but I don't know how to prove it! Thanks!
user141385's user avatar
2 votes
2 answers
90 views

Find the remainder of a division

Which is the remainder of the division $985^{423}:98$? That's what I have tried so far: Let $a=985,n=98$. Then $(a,n)=1$ and $\varphi(n)=42$. So, we have that $985^{42}\equiv 1 \pmod{98}$. Hence, $$\...
evinda's user avatar
  • 7,833
2 votes
1 answer
212 views

What is the smallest number k, such that $k^{2014}+2014$ is prime?

What is the smallest number k, such that $$k^{2014}+2014$$ is prime ? I checked upto $k= 24000$ and did not find a prime. Since the numbers do not grow very fast ($k=92204$ produces a $10 000$-...
Peter's user avatar
  • 84.6k
2 votes
2 answers
185 views

Let $p$ be the largest prime less than or equal to $n$. Is $n$ $\underline{<}$ $p^2$

Fix $n$ to be some positive integer greater than or equal to 2. Let $p$ be the largest prime less than or equal to $n$. Is $n$ $\underline{<}$ $p^2$?
singingmatt's user avatar
2 votes
2 answers
109 views

Does there exist an infinite sequence $p_0,p_1,p_2...$ of prime numbers such that $p_k=4 p_{k-1}\pm 1$

$k \in Z^+$ firstly we know that there exists infinetly many primes of the form $4n+1$ by FTA also we see that if we consider finite primes say to $n$ then the recursive formular can be expressed as:...
Jonas Kgomo's user avatar
  • 1,032
2 votes
1 answer
191 views

Proof on a conjecture involving $d(N)$

Let $d(n)$ denote the number of divisors of $n $. Then let $N$ be a number such that $d(N)$ divides $N$ . Also let $I= \frac{N}{d(N)}$ which is defined as the "Index of Beauty of $N$ ". Then prove ...
Shivam Patel's user avatar
  • 4,012
2 votes
2 answers
477 views

AIME number theory problem (unique factorization domains)

I'd greatly appreciate some help with the following problem, from a mock AIME I took. Compute the largest squarefree positive integer $n$ such that $\mathbb{Q}(\sqrt{-n})\cap \overline{\mathbb{Z}}$ ...
Ayesha's user avatar
  • 2,640
2 votes
1 answer
299 views

$n! =$ the product of consecutive integers. [duplicate]

Can $n!$ be the product of $k$ consecutive integers for $k > 1$? (Not including the degenerate cases such as when $k = 2$, then $1\cdot2 = 2!$ and $2\cdot 3 = 3!$, and so on.) I am asking not for $...
MT_'s user avatar
  • 19.6k
2 votes
1 answer
136 views

Between Mertens' theorems

It is well-known that $$ \sum_{p\le x}\frac{\log p}{p}=\log x+O(1) $$ and $$ \sum_{p\le x}\frac1p=\log\log x+M+o(1). $$ What is the order of $$ \sum_{p\le x}\frac{\sqrt{\log p}}{p} $$ ?
Charles's user avatar
  • 32.1k
2 votes
2 answers
857 views

Divisibility by 8 when converted in base 10

How many 7-digit numbers are there in binary(base-2)? How many of them are divisible by 8 when converted to base 10? My approach: In binary system, only two digits are there i.e. 0 and 1. So except ...
Walter White's user avatar
2 votes
1 answer
184 views

Limit of $\sum\frac{1}{p(\pi(n))}$

Let $p(n)$ be the nth prime and $\pi(n)$ the number of primes not exceeding n. I wonder if we can show that $$\tag{1} S = \sum_{n= 2}^k \frac{1}{ p (\pi (n))} \sim \log k. $$ We know by comparison ...
daniel's user avatar
  • 10.2k
2 votes
3 answers
11k views

Solving $ x^3 - 3x + \sqrt 2 = 0$

First of all, I'm going to use plain text for my formulas since I'm using a screen reading program, which makes it very difficult to read the highly non-standard format commonly used here, sorry about ...
Petr's user avatar
  • 31
2 votes
2 answers
752 views

Possible remainders when $3^{4n-2}+ 2^{6n-3} + 1$ is divied by $17$

The numbers in the form of $3^{4n-2}+ 2^{6n-3} + 1$, where $n$ is a positive integer, when divided by $17$, has possible remainders?
Zero's user avatar
  • 345
2 votes
3 answers
7k views

How to find all the primitive roots in $\mathbb{Z}/49\mathbb{Z}$.

I need to find all the primitive roots of 49. First note, $ ϕ(49) = 42 $ Is there an easier way to go about trying all numbers less than $42$ to find the primitive roots of $49$ if we already know ...
walt23's user avatar
  • 35
2 votes
2 answers
212 views

Quadratic Forms and Congruences

How does one prove (the non-trivial direction) that, for $n \in \mathbb{N}$, $x^2 + y^2 + z^2 = n$ solvable $\iff$ $x^2 + y^2 + z^2 \equiv n\ (m)$ solvable for all $m$? In particular, is there a ...
J.H.'s user avatar
  • 534
2 votes
2 answers
2k views

How to factor 30 digit number

I need to find the prime factorization of a number having 30 digits. I used the Pollard rho method but unfortunately it is not sufficient enough. It needs a more advanced prime factorization process....
palatok's user avatar
  • 357
2 votes
3 answers
144 views

Whether twin primes satisfy this one?

It seems that difference of squares of any twin primes $+1$ will always lead to number which might be a) A square of a twin prime b) Itself a twin prime $C$ = ($A^2$-$B^2$ )+$1$ ------> $(1)$ Where ...
Shan's user avatar
  • 597
2 votes
2 answers
94 views

Closed form way to express $a^1 + a^2 + a^3 + a^4 \cdots a^n$, given $a$ and $n$?

Is there a way to sum up consecutive powers like this when we know the value of $a$ using a closed form expression or do we actually have to add up each one? By closed form I mean in the same way ...
KaliMa's user avatar
  • 923
2 votes
4 answers
2k views

Whether any even number can be written as sum of odd no of primes?

Whether any even number can be written as sum of odd number of primes?(3,5,7.. primes) I know that Goldbach's conjecture state that a even number can be written as sum of two primes. D=A+B+C+...+n ...
Shan's user avatar
  • 597
2 votes
1 answer
128 views

Find prime numbers $a, b, c$ such that $a^b+c$ is a prime

Please help me find all $a, b, c \in \mathbb{P}$ such that $a^b+c$ is a prime Example I just can find: $2^3 + 5 = 13 \in \mathbb{P}$
Xeing's user avatar
  • 2,967
2 votes
1 answer
325 views

continued fraction expression for $\sqrt{2}$ in $\mathbb{Q_7}$

Hensel's lemma implies that $\sqrt{2}\in\mathbb{Q_7}$. Find a continued fraction expression for $\sqrt{2}$ in $\mathbb{Q_7}$
user46220's user avatar
  • 539
2 votes
4 answers
528 views

Exercise on Fundamentals of Divisibility: Factorization Domains

Set $R=\mathbb{Z}[\sqrt{10}]$. Show that in $R$ every element $\alpha\not=0$ is a product of irreducible elements, but $R$ is not a unique factorization domain. I have shown that $R$ is not a unique ...
Chris's user avatar
  • 421
2 votes
2 answers
97 views

numbers of subsets for a set $A$ for which the equation $x+y=2n+1$ hasn't solutions.

Find the numbers of subsets for the set: $\displaystyle A= \{1,2,\ldots,2n\}$ for which the equation $\displaystyle x+y=2n+1$ has not solutions. I have no idea. Thanks for your help.
Iuli's user avatar
  • 6,790
2 votes
4 answers
689 views

Is $x^y$ - $a^b$ divisible by $z$, where $y$ is large?

The exact problem I'm looking at is: Is $4^{1536} - 9^{4824}$ divisible by $35$? But in general, how do you determine divisibility if the exponents are large?
Takkun's user avatar
  • 433
2 votes
3 answers
243 views

Is 2 a quadratic residue modulo $r=\frac{p^m+1}{2}$?

Suppose that $$r=\frac{p^m+1}{2}$$ is a prime number, where $p$ is also prime. Does the equation $$p^{2}-2\equiv 0 \pmod{r}$$ have any solutions? Thanks in advance.
Sara's user avatar
  • 133
2 votes
2 answers
441 views

Is $n = k \cdot p^2 + 1$ necessarily prime if $2^k \not\equiv 1 \pmod{n}$ and $2^{n-1} \equiv 1 \pmod{n}$?

$p$ is an odd prime and $k$ is a positive integer. Let $n=k \cdot p^2+1$. If $2^k \not\equiv 1 \pmod n$ and $2^{n-1} \equiv 1 \pmod n$, is $n$ prime? If it is, why?
gkdffhpgdz's user avatar
2 votes
3 answers
1k views

Legendre Symbol Problem

I am doing revision for my number theory exam and I am stuck on the following question. Let $x$ be an even integer. Show that every prime divisor $p$ of $x^4 + 1$ satisfies $\big(\frac{-1}{p}\big)$ = ...
Anon's user avatar
  • 1,359
2 votes
2 answers
320 views

Proof that $ (a)^n \bmod n^2 = (a \bmod n)^n \bmod n^2$

Proof that $ (a)^n \bmod n^2 = (a \bmod n)^n \bmod n^2$ I did try a couple of examples and they do seem to work, but I just can't get why it works.
user996522's user avatar
2 votes
2 answers
397 views

Heuristic Proof of Hardy-Littlewood Conjecture for 3-term Arithmetic Progressions

The Hardy-Littlewood Conjecture for 3-term arithmetic progressions is that $$ \# \{ x,d \in \{1,\ldots,N\} \, | \, x,x+d,x+2d \text{ are all prime} \} \sim \frac{3}{2} \prod_{p > 2} \left(1+\...
fferic's user avatar
  • 61
2 votes
2 answers
714 views

If $a$ is even and $b$ is odd then $\gcd(2^{a}+1,2^{b}+1)=1$

How to prove that: If $a$ is even and $b$ is odd then $\gcd(2^{a}+1,2^{b}+1)=1$ This statement is generalization of the statement from my previous question. I have checked for many $(a,b)$ pairs ...
Pedja's user avatar
  • 12.9k
2 votes
3 answers
1k views

Proof: if $n=pq$ then $p-1\mid q-1$ and $q-1\mid p-1$

Now I'm asking my first question to understand a specific proof: Let $n=pq$ and $q,p \in \mathbb{P}$. Then we get $p-1\mid n-1$ and $q-1\mid n-1$, because there are prime integers mod $p$ and mod $q$....
ulead86's user avatar
  • 3,381
2 votes
2 answers
114 views

Modular functions and modular forms

A modular form of weight $k$ as a holomorphic function satisfying $f(\gamma z)=(cz+d)^kf(z)$ (automorphy relation of weight $k$) on the upper-half plane and also holomorphic at cusps. A modular ...
CO2's user avatar
  • 1,373
2 votes
1 answer
172 views

Missing pattern in solvable negative Pell equation

Considering the negative Pell equation $ x^2 - Dy^2 = -1 $, I know that a necessary condition for solvability is that $ D = a^2 + b^2$, with $ a,b $ positive integers. If I fix $ b = 1 $, NPE is ...
user967210's user avatar
2 votes
3 answers
54 views

Order of Odd Elements in $\mathbb{Z}_{2^n}$

I'm wondering if there is a way to calculate the order of odd numbers in the cyclic ring $\mathbb{Z}_{2^n}$. I found a paper that shows that for any odd $x$, $x^{2^{n-2}} \equiv 1 \mod{2^n}$, and that ...
Yoni Maltsman's user avatar
2 votes
2 answers
120 views

Writing a square as a sum of three non-zero squares in geometric progression

Let $k$ be a given positive integer. I want to solve the following system of Diophantine equations: $$\begin{cases} a^2 + b^2 + c^2 = k^2 \\ b^2 = ac \end{cases}$$ where $a, b, c \in \mathbb{N}$ are ...
Gauss's user avatar
  • 2,619
2 votes
1 answer
117 views

Relating the two errors terms in the prime number theorem

I would like to show that the two errors terms in the prime number theorem $\pi(x)-\frac{x}{\log{x}}$ and $\psi(x)-x$ are quite similar (or differing by something like a factor of $(\log{x})^{\...
Itachi's user avatar
  • 474
2 votes
4 answers
158 views

Find the mystery fraction

Question: There is a fraction with integer numerator and integer denominator, each smaller than $100$, whose decimal expansion begins with $0.11235\ldots$. Both the numerator and the denominator are ...
Crystal's user avatar
  • 31
2 votes
1 answer
79 views

Find three different number $m,n, k\in\mathbb{N}$ with $m, n, k\geq 3$ such that $mnk=2(mn+nk+mk)$

In my research, I need to solve the equation $mnk=2(mn+nk+mk)$ in natural number set, such that $n, m, k\in\mathbb{N}$ are different number and $n, m, k\geq 3$. I know that $(n, m, k)\in\{ (3, 7, 42)...
user479859's user avatar
  • 1,287
2 votes
1 answer
328 views

What is the general number of solutions of the equation $y^2=x^3-x\pmod{p}$?

What is the general number of solutions of the equation $y^2=x^3-x\pmod{p}$, where $p$ is a prime number and $p>3$?. Calculations suggest that the number of solutions to this equation is $p$ if $p\...
user avatar
2 votes
2 answers
93 views

Find $\sqrt{\frac{1}{2}-f(1)}+\dots+ \sqrt{\frac{1}{2}-f(99)}$

Define $f(n)=\sqrt[2]{n^4+\frac{1}{4}}-n^2.$ Find $$\sqrt{\frac{1}{2}-f(1)}+\dots+ \sqrt{\frac{1}{2}-f(99)}$$ I tried to simply $f(n).$ So rationalising, we get $$\sqrt{\frac{1}{2}-f(n)}\sqrt{\frac{1}{...
Raheel's user avatar
  • 1,607
2 votes
1 answer
143 views

Investigating $\sum \prod_{p\mid n}(1-\frac{1}{p^2})$ as $x\to\infty$

I'm investigating the behavior of the following function as $x\to \infty$: $$f(x):=\sum_{1\le n\le x}\frac{J_2(n)}{n^2}$$ where $J_k(n)$ is the Jordan's totient function $$J_k(n):=n^k\prod_{p\mid n}(1-...
Tianlalu's user avatar
  • 5,187
2 votes
3 answers
304 views

Find all pairs of primes $(p, q)$ such that $p^2 + 6pq + q$ is a perfect square [closed]

Find all pairs of primes $(p, q)$ such that $p^2 + 6pq + q$ is a perfect square. This is a problem I encountered a month ago. I remember having had multiple attempts, but they didn't seem to lead ...
StriveOnlyToFail's user avatar

1
286 287
288
289 290
820